Jump to content

poles of an electromagnet question


DandelionTheory

Recommended Posts

when current passes through a conductor loop it makes a magnetic field within the loop according to this site:

http://hyperphysics.phy-astr.gsu.edu/hbase/magnetic/curloo.html

according to the following site, the magnetic field direction of a current carrying wire goes from north to south with respect to the right hand rule:

https://nationalmaglab.org/education/magnet-academy/watch-play/interactive/magnetic-field-around-a-wire-i

in the following picture i drew i attempt to recreate this with an iron core, and represent the resulting respective pole in the color magenta:

 

magnet_fig1.thumb.jpg.a86ecbacff74e77e3f96323778c04464.jpg

is my recreation of the general effect correct?

-DT

Edited by DandelionTheory
Link to comment
Share on other sites

i have drawn another picture to ask a specific question visually because of the new shape of the core.

in the new picture (labeled as fig 2), i show the iron core as a capital I shape. are the indicated poles correct?

magnet_fig2.thumb.jpg.c88b9c056aa7f3196652c7e33b4fb17c.jpg

what im doing is reshaping the magnet to reorient the dipoles.

ill show you in the next one if this looks right, i know its tedious but you will have a better chance with fig 4.

Link to comment
Share on other sites

17 hours ago, studiot said:

You have drawn 4 poles, not 2.

Why do you think magnetic cores are not made in this way; (the end cheecks of a coil winding bobbin are made of non magnetic material)?

im attempting to understand what i can and cannot do; why i can and cannot do them, and what kind of responses i get by asking them.

because i know its not normal, im trying to see if its correct.

because my next question is can i move the dipoles to another place? and then how do i calculate the sum of forces over time on all structures in the system of figure 4?

 

magnet_fig4.jpg

Edited by DandelionTheory
Link to comment
Share on other sites

it appears i have found something, but im still not too sure ive applied it correctly.

in attempting to calculate the B field interacting with the current, i found a resource in calculating B field between the air gaps of an iron core transformer:

https://en.wikipedia.org/wiki/Electromagnet

in section "Magnetic field created by a current" it explains how to calculate the B field of an iron core magnet with an air gap. i would then use the Lorenz force to calculate the force experienced by the current.

equasion_without_B.thumb.jpg.c891d201bdf91cb54fe2c4fb038abfbf.jpg

 

from what i can tell, the placement of a current next to its own dipoles is a weird concept. I'm assuming this produces a momentum change in the positive Y direction, but i can be wrong.

ps, im looking for a tutor in math.

Edited by DandelionTheory
Link to comment
Share on other sites

  • 1 month later...
On 12/22/2019 at 12:26 AM, DandelionTheory said:

it appears i have found something, but im still not too sure ive applied it correctly.

Since the current seems to be drawn as a loop it's direction is into the paper on one side and out of the paper on the other side. Since the B-field seems to have same direction in both of the air gaps and the current have opposite directions you have to take this into account in the calculations. 

On 12/22/2019 at 12:26 AM, DandelionTheory said:

I'm assuming this produces a momentum change in the positive Y direction, but i can be wrong.

 Your assumption is wrong. There cannot be a net force on the system as it is configured in the picture above. 

 

Link to comment
Share on other sites

On 1/25/2020 at 2:43 PM, Ghideon said:

Since the current seems to be drawn as a loop it's direction is into the paper on one side and out of the paper on the other side. Since the B-field seems to have same direction in both of the air gaps and the current have opposite directions you have to take this into account in the calculations. 

The lorentz force is applied to a current sin theta to the magnetic field it's in, perpendicular to it's velocity. Period. Where did that not apply?

I didn't account for forces on the dipoles

Edited by DandelionTheory
Link to comment
Share on other sites

1 hour ago, Ghideon said:

B-field direction (green): both arrows should have same direction. 

Because the B field is the same on both sides? No it's not. Look again.

North to south. As the current rotates about the center axis the direction of the B field doesn't change. Did you rotate your hand with the current as it went around the south pole? Lol

48430700_20191221_1458373.thumb.jpg.831fc5d7b55be4cdd426f611b24aacc6.jpg

Edited by DandelionTheory
Link to comment
Share on other sites

24 minutes ago, DandelionTheory said:

Look again.

Your green vectors seems ok, I was confused by different types of transformers! A shell type transformer would have the magnetic flux in the direction shown by green arrows. Sorry for confusion.

It does not affect this comment:

On 12/22/2019 at 12:26 AM, DandelionTheory said:

I'm assuming this produces a momentum change in the positive Y direction, but i can be wrong.

 Your assumption is wrong. There cannot be a net force on the system as it is configured in the picture above. 

 

 

Edited by Ghideon
Link to comment
Share on other sites

8 minutes ago, DandelionTheory said:

Because you can't assume.

I am applying the basic laws of Newton. 
 

9 minutes ago, DandelionTheory said:

I mean, you claimed there is no net force on the system. Is that because of the opposite force on the magnetic dipoles due to I?

This is speculations: provide supporting evidence for your claim. I do not have to provide evidence for Newton's laws.

 

image.png

Link to comment
Share on other sites

18 minutes ago, Ghideon said:

I do not have to provide evidence for Newton's laws.

So you just assume they apply to everything you think, and there is no way around any of it even with addition or cancelation. Cool that's not my problem. I'm learning buddy, could you teach me or do you just assume I'm trying to write a paper.... Because this isn't a scientific paper man. This is an idea to do a thing. Weather it applies to the whole I acknowledged that it doesn't account for force on the dipoles. Is it a lifter? Did I say it had mass or how much force or which way gravity was? No? Then how could it do what you said?

You need to show your work or you don't have a standpoint. I'm trying to show mine, but you assume alot and it's insulting. Could you maybe give me something to look up because you say a bunch and complain a lot but I haven't seen your reasoning just claims. Could YOU draw a diagram?

Reactions have a magnitude.

So is my calculation on I true or not

Edited by DandelionTheory
Link to comment
Share on other sites

11 minutes ago, DandelionTheory said:

I'm trying to show mine, buy you assume shit alot it's insulting.

Ok. 

On 12/22/2019 at 12:26 AM, DandelionTheory said:

I'm assuming this produces a momentum change in the positive Y direction, but i can be wrong.

Can you provide a calculation of the amount of momentum you expect? 

 

13 minutes ago, DandelionTheory said:

So is my calculation on I true or not

Does the dot in the calculations imply dot product or cross product for the vectors? 

Link to comment
Share on other sites

5 minutes ago, Ghideon said:

Does the dot in the calculations imply dot product or cross product for the vectors?

So you are not familiar with the lorentz force?

 

5 minutes ago, Ghideon said:

Can you provide a calculation of the amount of momentum you expect?

No. That's not the question. The question is 

 

On 12/21/2019 at 3:26 PM, DandelionTheory said:

I'm assuming this produces a momentum change in the positive Y direction, but i can be wrong.

If you didn't get "the current experiences a momentum change in the y direction" I can't help you. That's a loss in translation or something else.

Link to comment
Share on other sites

6 hours ago, DandelionTheory said:

So you are not familiar with the lorentz force?

I am familiar with Lorentz force. 

This is the formula:

image.png.bb372d4d3801e469c6a9653d5cecbc12.png

Your formula:

eq.png.1d517cca6f9a63ec2a7d054ac8671eb3.png

Vector dot product and vector cross product is not the same. So the answer to:

7 hours ago, DandelionTheory said:

So is my calculation on I true or not

The answer is that your calculation is incorrect. But I asked "if the dot in the calculations imply dot product or cross product for the vectors" in case it was just a typo you made. 

 

6 hours ago, DandelionTheory said:

If you didn't get "the current experiences a momentum change in the y direction" I can't help you. That's a loss in translation or something else.

I think we had this discussion in  another thread. (Cherry) picking one concept from physics, such as Lorentz Force, does not remove the need to account for other concepts or laws, such as conservation of momentum or an equal and opposite force . 

Edited by Ghideon
format and clarification
Link to comment
Share on other sites

9 hours ago, Ghideon said:

I think we had this discussion in  another thread. (Cherry) picking one concept from physics, such as Lorentz Force, does not remove the need to account for other concepts or laws, such as conservation of momentum or an equal and opposite force 

Never said it wasn't opposite, I'm framing a picture of a force. You assume other things affect this force, cool, I want to not look at that hole you put your mind in for a minute and maybe explore weird things we see. Is it incorrect because you see conflicting forces? that's not what I'm pointing out genius. The fact that you cannot comprehend a basic ass idea like this without interception of opposing ideas makes you incapable of seeing solutions to specific problems. "Checking people's work" is only beneficial to you if you know what you're correcting and you do it correctly, you are not in a patent office.

Mansplaining the lorentz force makes you look stupid due to the fact that I've been trying to get you to reference it for a few weeks and you asked if it was a dot product. I know it, my slight of hand in notation shouldn't phase a someone who knows the equations. If you don't know what I'm talking about then don't respond.

Link to comment
Share on other sites

7 hours ago, DandelionTheory said:

Mansplaining the lorentz force makes you look stupid due to the fact that I've been trying to get you to reference it for a few weeks and you asked if it was a dot product. I know it, my slight of hand in notation shouldn't phase a someone who knows the equations. If you don't know what I'm talking about then don't respond.

!

Moderator Note

You need to recalibrate your attitude. Lashing out at the people taking the time to answer your questions and giving the feedback you ask for is unacceptable.

This is not an invitation to respond in this thread and try and defend your actions.

 
Link to comment
Share on other sites

 

9 hours ago, DandelionTheory said:

I know it, my slight of hand in notation shouldn't phase a someone who knows the equations.

You asked if your calculation is correct. I wanted to make sure that your calculation correctly expressed what you intended. 

Back to the calculation. Note that the current in the cable generates it's own magnetic field. But we can probably neglect that in this case unless you intend to have a large current and a weak magnetic field. And if you want to calculate the force on the cable due to the magnetic field then the E field can be neglected. If you know the dimensions it might be easier/more practical to use length and current instead of charge "q" and velocity "v". But that is just details, the formula seems ok for the force on the current carrying cable due to magnetic field.

 

9 hours ago, DandelionTheory said:

"Checking people's work" is only beneficial to you if you know what you're correcting and you do it correctly, you are not in a patent office.

Incorrect. The purpose of the speculations section is "checking peoples work" or "have our idea checked". Unfortunately that means you are sometimes stuck with amateurs like me, the professionals seems not t be very interested. An alternative is to write a scientific paper and have it peer reviewed by professionals working for a journal.

 

Link to comment
Share on other sites

2 minutes ago, Ghideon said:

The purpose of the speculations section is "checking peoples work"

I didn't put it here. It was moved. I made statements with math and it was still moved to speculation. 

For an op to ask questions about physics in a classical physics forum is not off base, but to assume any point being made is base for the whole post to be sent to speculation... Best move. I don't know what the forum is for if not to ask questions, and for any point to be assumed speculation isn't a forum it's gatekeeping.

6 minutes ago, Ghideon said:

Back to the calculation. Note that the current in the cable generates it's own magnetic field. But we can probably neglect that in this case unless you intend to have a large current and a weak magnetic field. And if you want to calculate the force on the cable due to the magnetic field then the E field can be neglected. If you know the dimensions it might be easier/more practical to use length and current instead of charge "q" and velocity "v". But that is just details, the formula seems ok for the force on the current carrying cable due to magnetic field.

Thank you. Please allow me to aggregate the needed variables.

Link to comment
Share on other sites

Guest
This topic is now closed to further replies.
×
×
  • Create New...

Important Information

We have placed cookies on your device to help make this website better. You can adjust your cookie settings, otherwise we'll assume you're okay to continue.